You are on page 1of 67

POLITY

Chapter 1: Historical Background

Q.1) Which of the following acts recognized for the first time, the political and administrative
functions of the East India Company?

a) Regulating Act 1773


b) Pitts India Act 1784
c) Charter Act of 1833
d) Government of India Act 1858

Q.1) Solution (a)

Regulating act of India 1773 was a landmark decision as the British Government for the first
time interfered into affairs of India.

This act is of great constitutional importance as

(a) it was the first step taken by the British Government to control and regulate the affairs
of the East India Company in India;
(b) it recognised, for the first time, the political and administrative functions of the
Company; and
(c) it laid the foundations of central administration in India.

Q.2) Consider the following pairs.

Constitutional developments Significance

1. Charter Act of 1833 Indian Legislative Council

2. Charter Act of 1853 Final step towards centralization

3. Government of India Act 1919 Introduction of responsible government in


India

4. Government of India Act 1935 Introduction of responsible government in


provinces

www.iasbaba.com Page 1
POLITY

Which of the pairs given above are correctly matched?

a) 1 and 3 only
b) 2 and 3 only
c) 3 and 4 only
d) All the above

Q.2) Solution (c)

Charter Act of 1833

 It redesignated the Governor-General of Bengal as the Governor-General of India. Under


this provision Lord William Bentinck became the first Governor-General of India.
 It deprived the Governors of Bombay and Madras of their legislative powers. For the
first time, the Governor-General's Government was known as the 'Government of India'
and his council as the 'India Council'. The Governor-General and his executive council
were given exclusive legislative powers for the whole of British India.

Charter Act of 1853

 It separated for the first time, the legislative and executive functions of the governor-
general Council. It provided for the addition of six members called legislative councilors
to the council. In other words, it established a separate Governor’s General legislative
council which came to be known as Indian legislative Council.

Government of India Act 1919

 In November 1917, Montague had visited India to ascertain views from all sections of
polity including talks with Gandhi and Jinnah. A detailed report on Constitutional
Reforms in India {Mont-Ford Report} was published on 8th July 1918. This report
became the basis of Government of India Act 1919. Key features of this report were as
follows:
 Increasing association of Indians in every branch of administration.
 Gradual development of self-governing institutions with a view to the progressive
realisation of responsible government in India as an integral part of the British empire.
 Progress towards responsible government in successive stages.

Government of India Act 1935.

www.iasbaba.com Page 2
POLITY

 The most remarkable feature of the Act was the provincial autonomy. With the abolition
of Dyarchy at provinces, the entire provincial administration was instructed to the
responsible ministers who were controlled and removed by the provincial legislatures.
 The provincial autonomy means two things. First, the Provincial Governments were
wholly, responsible to the provincial legislatures and secondly, provinces, were free
from outside control and interference in a large number of matters. Thus, in the
provincial sphere, the Act of 1935 made a fundamental departure from the act of 1919.

Q.3) Which among the following British India Acts ended the system of double government
by abolishing the Board of Control and Court of Directors?

a) Charter Act of 1853


b) Government of India Act of 1858
c) Indian Councils Act of 1861
d) Indian Councils Act of 1892

Q.3) Solution (b)

Government of India Act of 1858 - The act known as the Act for the Good Government of India,
abolished the East India Company, and transferred the powers of government, territories and
revenues to the British Crown.

It also ended the system of double government by abolishing the Board of Control and Court of
Directors.

Do you know?

Other Features of the Government of India Act of 1858

1. It provided that India henceforth was to be governed by, and in the name of, Her
Majesty. It changed the designation of the Governor-General of India to that of Viceroy
of India. He (viceroy) was the direct representative of the British Crown in India. Lord
Canning thus became the first Viceroy of India.
2. It created a new office, Secretary of State for India, vested with complete authority and
control over Indian administration. The secretary of state was a member of the British
cabinet and was responsible ultimately to the British Parliament.
3. It established a 15-member Council of India to assist the secretary of state for India. The
council was an advisory body. The secretary of state was made the chairman of the
council.

www.iasbaba.com Page 3
POLITY

4. It constituted the secretary of state-in-council as a body corporate, capable of suing and


being sued in India and in England.

Q.4) Consider the following provisions under Government of India 1935 act:

1. It abolished diarchy at the center and adopted it in provinces.


2. Abolished council of India, which was established in GOI act 1858 to assist secretary of
state
3. Provided for the establishment of federal court

Which of the above statements are correct?

a) 1 and 2
b) 2 and 3
c) 1 and 3
d) All of the above

Q.4) Solution (b)

Government of India Act of 1935

The Act marked a second milestone towards a completely responsible government in India. It
was a lengthy and detailed document having 321 Sections and 10 Schedules.
Features of the Act
1. It provided for the establishment of an All-India Federation consisting of provinces and
princely states as units. The Act divided the powers between the Centre and units in terms of
three lists—Federal List (for Centre, with 59 items), Provincial List (for provinces, with 54 items)
and the Concurrent List (for both, with 36 items). Residuary powers were given to the Viceroy.
However, the federation never came into being as the princely states did not join it.

2. It abolished dyarchy in the provinces and introduced ‘provincial autonomy’ in its place. The
provinces were allowed to act as autonomous units of administration in their defined spheres.
Moreover, the Act introduced responsible governments in provinces, that is, the governor was
required to act with the advice of ministers responsible to the provincial legislature. This came
into effect in 1937 and was discontinued in 1939.

3. It provided for the adoption of dyarchy at the Centre. Consequently, the federal subjects
were divided into reserved subjects and transferred subjects. However, this provision of the Act
did not come into operation at all.

www.iasbaba.com Page 4
POLITY

4. It introduced bicameralism in six out of eleven provinces. Thus, the legislatures of Bengal,
Bombay, Madras, Bihar, Assam and the United Provinces were made bicameral consisting of a
legislative council (upper house) and a legislative assembly (lower house). However, many
restrictions were placed on them.
5. It further extended the principle of communal representation by providing separate
electorates for depressed classes (scheduled castes), women and labour (workers).
6. It abolished the Council of India, established by the Government of India Act of 1858. The
secretary of state for India was provided with a team of advisors.
7. It extended franchise. About 10 per cent of the total population got the voting right.
8. It provided for the establishment of a Reserve Bank of India to control the currency and
credit of the country.
9. It provided for the establishment of not only a Federal Public Service Commission but also a
Provincial Public Service Commission and Joint Public Service Commission for two or more
provinces.
10. It provided for the establishment of a Federal Court, which was set up in 1937.

Q.5) Consider the features given below and choose the correct answer:

1. This Act provided for the first time for the association of Indians with the executive
Councils of the Viceroy and Governors.
2. This Act introduced a system of communal representation for Muslims by accepting the
concept of ‘separate electorate’.
3. This Act retained official majority in the Central Legislative Council but allowed the
provincial legislative councils to have non-official majority.

The above important features are part of –

a) Indian Councils Act of 1861


b) Indian Councils Act of 1892
c) Indian Councils Act of 1909
d) Government of India Act of 1919

Q.5) Solution (c)

Features of the Act of 1909 also known as Morley-Minto Reforms:

1. It considerably increased the size of the legislative councils, both Central and provincial.

www.iasbaba.com Page 5
POLITY

2. It retained official majority in the Central Legislative Council but allowed the provincial
legislative councils to have non-official majority.
3. It enlarged the deliberative functions of the legislative councils at both the levels. For
example, members were allowed to ask supplementary questions, move resolutions on
the budget, and so on.
4. It provided (for the first time) for the association of Indians with the executive Councils
of the Viceroy and Governors.
5. It introduced a system of communal representation for Muslims by accepting the
concept of ‘separate electorate’.
6. It also provided for the separate representation of presidency corporations, chambers of
commerce, universities and zamindars.

Chapter 2: Making of the Constitution

Chapter 3: Salient Features of the Constitution

Q.6) Apart from the making of Constitution, which of the following functions were performed
by the Constituent Assembly?

1. Ratified India’s membership of the Commonwealth


2. Elected Dr. Rajendra Prasad as the first President of India
3. Adopted India’s National Anthem and National Song
4. Adopted the National Flag of India

Select the code from the following:

a) 1,2 and 3
b) 2,3 and 4
c) 1,3 and 4
d) All of the above

Q.6) Solution (d)

In addition to the making of the Constitution and enacting of ordinary laws, the Constituent
Assembly also performed the following functions:

 It ratified the India’s membership of the Commonwealth in May 1949.


 It adopted the national flag on July 22, 1947.

www.iasbaba.com Page 6
POLITY

 It adopted the national anthem on January 24, 1950.


 It adopted the national song on January 24, 1950.
 It elected Dr Rajendra Prasad as the first President of India on January 24, 1950.

Q.7) With reference to the Constitution of India, consider the following statements:

1. The Constitution Day is celebrated to commemorate the commencement of


constitution.
2. Republic Day is celebrated to commemorate the adoption of constitution.

Which of the above statement is/are correct?

a) 1 only
b) 2 only
c) Both 1 and 2
d) Neither 1 nor 2

Q.7) Solution (d)

Constitution Day (National Law Day), also known as Samvidhan Divas, is celebrated in India on
26 November every year to commemorate the adoption of Constitution of India. On 26
November 1949, the Constituent Assembly of India adopted the Constitution of India, and it
came into effect on 26 January 1950.

The Constitution was adopted by the Indian Constituent Assembly on 26 November 1949, and
came into effect on 26 January 1950 with a democratic government system, completing the
country's transition towards becoming an independent republic. 26 January was chosen as the
Republic day because it was on this day in 1930 when Declaration of Indian Independence
(Purna Swaraj) was proclaimed by the Indian National Congress as opposed to
the Dominion status offered by British Regime.

Do you know?

 The Beating Retreat ceremony is held after officially denoting the end of Republic Day
festivities. It is conducted on the evening of 29 January, the third day after the Republic
Day. It is performed by the bands of the three wings of the military, the Indian
Army, Indian Navy and Indian Air Force. The venue is Raisina Hill and an adjacent
square, Vijay Chowk, flanked by the North and South block of the Rashtrapati

www.iasbaba.com Page 7
POLITY

Bhavan (President's Palace) towards the end of Rajpath. The Chief Guest of the function
is the President of India who arrives escorted by the (PBG), a cavalry unit.

Q.8) Which of the following statements are correct regarding ‘Objectives Resolution’?

1. The resolution defined the aims of Constituent Assembly.


2. It was moved by Pt Nehru and adopted unanimously by INC in 1931.
3. The modified form of it forms present day Preamble of Indian Constitution.

Select the code from following:

a) 1 and 2
b) 2 and 3
c) 1 and 3
d) All of the above

Q.8) Solution (c)

Before the framing of the constitution started, an Objectives Resolution (the resolution that
defined the aims of the Assembly) was moved by Nehru in 1946. This resolution enshrined the
aspirations and values behind the Constitution making. On the basis of the Objectives
Resolution, India’s Constitution gave institutional expression to the fundamental commitments:
equality, liberty, democracy, sovereignty and a cosmopolitan identity. This made the moral
commitment to establish a government that will fulfill the many promises that the nationalist
movement held before the people of India.

In its modified form it was made the preamble of Indian Constitution.

Q.9) Which among the following is/are the feature common to both the Indian Federation
and the American Federation?

1. Three lists in the Constitution


2. A federal supreme court to interpret the Constitution
3. Single citizenship
4. Appointment of state governors by the Centre

Choose correct option:

a) 1 and 2 only
b) 1, 2 and 4 only

www.iasbaba.com Page 8
POLITY

c) 2 only
d) 2 and 3 only

Q.9) Solution (c)

‘A federal supreme court to interpret the Constitution’ is the only feature common to both the
Indian Federation and the American Federation. Hence option (c) is correct answer.

‘Appointment of state governors by the Centre’ feature was borrowed from Canadian
Constitution.

‘Single citizenship’ feature was borrowed from the British Constitution. Federal states like US,
Switzerland and Australia have dual citizenship

Q.10) Which Schedule of the Constitution deals with Allocation of seats in the House of
People?

a) Schedule Two

www.iasbaba.com Page 9
POLITY

b) Schedule Four
c) Schedule Three
d) None

Q.10) Solution (d)

None of the Schedules listed in the Constitution deals with Allocation of seats in the House of
People (Lok Sabha)

Followings are the schedules in Constitution of India

First
Schedule  Names of the States and names of Union Territories (UTs)

Second  Salary and allowances of President, Governors, Chief Judges, Judges of


Schedule High Court and Supreme court, Comptroller and Auditor General
Third
Schedule  Forms of Oaths and affirmations

Fourth  Allocation of seats in the Rajya Sabha to the states and the union
Schedule territories.

Fifth
Schedule  Administration and control of scheduled areas and tribes

Sixth  Provisions relating to the administration of tribal areas in the states of


Schedule Assam, Meghalaya, Tripura and Mizoram

 Division of powers
Seventh o Union List (for central Govt) 97 Subjects.
Schedule o States List (for State Govt) 66 subjects
o Concurrent List (for both Union and States) 47 subjects.

 List of 22 languages of India recognized by Constitution


Eighth
1. Assamese 2. Bengali 3. Gujarati
Schedule
4. Hindi 5. Kannada 6. Kashmiri

www.iasbaba.com Page 10
POLITY

7. Manipuri 8. Malayalam 9. Konkani

10. Marathi 11. Nepali 12. Oriya

13. Punjabi 14. Sanskrit 15. Sindhi

16. Tamil 17. Telugu 18. Urdu

19. Santhali 20. Bodo 21. Maithili

22. Dogri

 Sindhi was added in 1967 by 21 Amendment


 Konkani, Manipuri ad Nepali were added in 1992 by 71 amendment
Santhali, Maithili, Bodo and Dogri were added in 2003 by 92 amendment
 Contains Acts and Regulations dealing with land reforms and abolition of
the zamindari system.
 This schedule was added by the 1st Amendment (1951) to protect the
Ninth
laws included in it from judicial scrutiny on the ground of violation of
Schedule
fundamental rights.
 However, in 2007, the Supreme Court ruled that the laws included in this
schedule are now open to judicial review.

Tenth  Added by 52nd amendment in 1985. Contains provisions of


Schedule disqualification of grounds of defection
Eleventh
Schedule  By 73rd amendment in 1992. Contains provisions of Panchayati Raj.

Twelfth  By 74th amendment in 1992. Contains provisions of Municipal


Schedule Corporation.

4. Preamble of the Constitution

Q.11) Consider the following statements regarding the Preamble of Indian Constitution:

www.iasbaba.com Page 11
POLITY

1. In Keshavananda Bharti case, Supreme Court held that Preamble is not a part of the
Constitution.
2. The Preamble is neither a source of power to legislature nor a prohibition upon the
powers of legislature.
3. It is justiciable, that is, its provisions are enforceable in courts of law.

Which of the above statements are correct?

a) 1 and 2
b) 2 and 3
c) 1 and 3
d) All of the above

Q.11) Solution (a)

One of the controversies about the Preamble is as to whether it is a part of the Constitution or
not.

In the Berubari Union case (1960), the Supreme Court said that the Preamble shows the general
purposes behind the several provisions in the Constitution, and is thus a key to the minds of the
makers of the Constitution. Further, where the terms used in any article are ambiguous or
capable of more than one meaning, some assistance at interpretation may be taken from the
objectives enshrined in the Preamble. Despite this recognition of the significance of the
Preamble, the Supreme Court specifically opined that Preamble is not a part of the
Constitution.

In the Kesavananda Bharati case (1973), the Supreme Court rejected the earlier opinion and
held that Preamble is a part of the Constitution. It observed that the Preamble is of extreme
importance and the Constitution should be read and interpreted in the light of the grand and
noble vision expressed in the Preamble. In the LIC of India case18 (1995) also, the Supreme
Court again held that the Preamble is an integral part of the Constitution.

Like any other part of the Constitution, the Preamble was also enacted by the Constituent
Assembly, but, after the rest of the Constitution was already enacted. The reason for inserting
the Preamble at the end was to ensure that it was in conformity with the Constitution as
adopted by the Constituent Assembly. While forwarding the Preamble for votes, the president
of the Constituent Assembly said, ‘The question is that Preamble stands part of the
Constitution’19. The motion was then adopted. Hence, the current opinion held by the
Supreme Court that the Preamble is a part of the Constitution, is in consonance with the
opinion of the founding fathers of the Constitution.

www.iasbaba.com Page 12
POLITY

However, two things should be noted:

1. The Preamble is neither a source of power to legislature nor a prohibition upon the powers
of legislature.
2. It is non-justiciable, that is, its provisions are not enforceable in courts of law.

Q.12) Which of the following statements is/are true in regard to the Preamble of India?

1. The term ‘justice’ in the Preamble embraces social, economic and political justice.
2. The ideal of justice – social, economic and political – in our Preamble has been taken
from the American Revolution.
3. The ideals of liberty, equality and fraternity in our Preamble have been taken from the
French Revolution.

Select the appropriate code:

a) 1 and 2 only
b) 1 and 3 only
c) 2 and 3 only
d) All of the above

Q.12) Solution (b)

The term ‘justice’ in the Preamble embraces all the three – social, economic and political
justice.

The ideal of justice—social, economic and political— in our Preamble has been taken from the
Russian Revolution (1917).

The ideals of liberty, equality and fraternity in our Preamble have been taken from the French
Revolution.

Q.13) The ‘Fraternity’ mentioned in the preamble refers to the common brotherhood of all
citizens. The constitution promotes fraternity through which of the following?

1. Single citizenship
2. Fundamental rights
3. Fundamental duties
4. Directive principles of state policy

Select the correct answer using the codes below.

www.iasbaba.com Page 13
POLITY

a) 1 only
b) 2 and 3 only
c) 1 and 3 only
d) 1, 2 and 3 only

Q.13) Solution (c)

Fraternity means a sense of brotherhood. The Constitution promotes this feeling of fraternity
by system of single citizenship. Also, the Fundamental Duties(Article-51A) say that I shall be the
duty of every citizen of India to promote harmony and the spirit of common brotherhood
amongst all the people of India transcending religious, linguistic, regional, or sectional
diversities.

Do you know?

 The terms liberty, equality and fraternity in the Preamble are taken from French
revolution (1789-1799) and the ideal of justice social, economic and political from that
of Russian revolution (1917).

THINK!

 Preamble is the key to the minds of the makers of Constitution.

Q.14) Consider the following statements about the Preamble of the Constitution.

1. It is not justiciable in nature.


2. It cannot be amended.
3. It can override specific provisions of the constitution.
4. It has been a source of power to the executive.

Select the correct answer using the codes below.

a) 1 only
b) 2 and 3 only
c) 1 and 3 only
d) All the above

Q.14) Solution (a)

The Preamble is non-justiciable, that is, its provisions are not enforceable in courts of law.

www.iasbaba.com Page 14
POLITY

In Kesavanand Bharti case the supreme Court held that the Preamble can be amended, subject
to the condition that no amendment is done to the basic features.

The Preamble cannot override any of the provisions of the constitution, because it is neither a
source of power to legislature nor a prohibition upon the powers of legislature.

Do you know?

 The Preamble is amended only once so far, in 1976, by the 42nd Constitutional
amendment act, which added three new words socialist, secular and integrity.

Q.15) The Preamble reveals which among the following ingredients or components -

1. Source of authority of the Constitution


2. Nature of Indian State
3. Objectives of the Constitution
4. Date of adoption of the Constitution

Choose correct option:

a) 2 and 3 only
b) 2, 3 and 4 only
c) 1, 3 and 4
d) 1, 2, 3 and 4

Q.15) Solution (d)

The Preamble reveals four ingredients or components:

1. Source of authority of the Constitution: The Preamble states that the Constitution
derives its authority from the people of India.
2. Nature of Indian State: It declares India to be of a sovereign, socialist, secular
democratic and republican polity.
3. Objectives of the Constitution: It specifies justice, liberty, equality and fraternity as the
objectives.
4. Date of adoption of the Constitution: It stipulates November 26, 1949 as the date.

www.iasbaba.com Page 15
POLITY

Chapter 5: Fundamental Rights

Q.16) Which of the following is known ‘Conscience of the Constitution’ by Granville Austin?

1. Fundamental Rights
2. DPSPs
3. Fundamental Duties
4. Parliament

Select the code from below:

a) 1 and 2
b) 2 and 3
c) 1,2 and 3
d) All of the above

Q.16) Solution (a)

The Directive Principles along with the Fundamental Rights contain the philosophy of the
Constitution and is the soul of the Constitution. Granville Austin has described the Directive
Principles and the Fundamental Rights as the ‘Conscience of the Constitution’

Q.17) Which of the following Fundamental Rights are available only to Indian Citizens?

1. No discrimination on the basis of religion, race, caste, sex or place of birth.


2. Equal opportunity in the matters of Public employment
3. Freedom of Speech and expression
4. Protection of life and personal Liberty

Select the code from below:

a) 2 only
b) 1, 2 and 3
c) 1 and 2
d) All of the above

Q.17) Solution (b)

www.iasbaba.com Page 16
POLITY

Following Fundamental Rights are available only to citizens and not to foreigners:

 Prohibition of discrimination on grounds of religion, race, caste, sex or place of birth


(Article 15).
 Equality of opportunity in matters of public employment (Article 16).
 Six basic freedoms subject to reasonable restrictions (Article 19).
 Protection of language, script and culture of minorities (Article 29).
 Right of minorities to establish and administer educational institutions (Article 30).

Q.18) Consider the following statements:

1. Article 20 deals with protection against arrest and detention in certain cases.
2. The striking feature of the Article 20 is that it can’t be suspended during an emergency
period.

Which of the statements given above is/are correct?

a) 1 only
b) 2 only
c) Both 1 and 2
d) Neither 1 nor 2

Q.18) Solution (b)

Statement 1 is wrong as Article 22 deals with ‘Protection against arrest and detention in certain
cases’ (not Article 20). Article 20 deals with ‘Protection in respect of conviction for offenses’.

Article 20 {Protection in respect of conviction for offenses}

 Ex Post Facto Legislation: No person shall be convicted of any offence except for
violation of a law in force at the time of the commission of the act charged as an
offence, nor be subjected to a penalty greater than that which might have been inflicted
under the law in force at the time of the commission of the offence.
 Immunity from Double Punishment: No person shall be prosecuted and punished for
the same offence more than once.
 Immunity from Self-Incrimination: No person accused of any offence shall be compelled
to be a witness against himself.

www.iasbaba.com Page 17
POLITY

Statement 2 is correct. The striking feature of the Article 20 is that it can’t be suspended during
an emergency period.

Q.19) Which among the following are true with regard to Fundamental Rights?

1. They help to promote the ideal of political democracy


2. They help to prevent the establishment of an authoritarian and despotic rule in the
country
3. They operate as limitations on the tyranny of the executive and arbitrary laws of the
legislature
4. They aim at establishing ‘a government of laws and not of men’

Choose correct answer:

a) 2 and 3
b) 1, 2 and 3
c) 2, 3 and 4
d) 1, 2, 3 and 4

Q.19) Solution (d)

The Fundamental Rights are meant for promoting the ideal of political democracy. They prevent
the establishment of an authoritarian and despotic rule in the country, and protect the liberties
and freedoms of the people against the invasion by the State.

They operate as limitations on the tyranny of the executive and arbitrary laws of the legislature.
In short, they aim at establishing ‘a government of laws and not of men’.

Hence, all the given statements are correct.

Q.20) Bachan Singh vs. State of Punjab (1980) dealt with –

a) Witness Protection Law in India


a) Khalistan movement or Sikh separatist movement
b) More autonomy to Punjab on the model of Jammu and Kashmir
c) Capital punishment

www.iasbaba.com Page 18
POLITY

Q.20) Solution (d)

The Constitution Bench judgment of Supreme Court of India in Bachan Singh vs. State of
Punjab (1980) made it very clear that Capital punishment in India can be given only in rarest of
rare cases.

In other words, the Supreme Court of India ruled that the death penalty should be imposed
only in "the rarest of rare cases."

The Supreme Court in Mithu vs. State of Punjab struck down Section 303 of the Indian Penal
Code, which provided for a mandatory death sentence for offenders serving a life sentence.

Q.21) Which among the following are protected as an intrinsic part of the right to life and
personal liberty under Article 21 and as a part of the freedoms guaranteed by Part III of the
Constitution?

1. Live-in Relationships
2. Right to privacy
3. Right to electricity

Choose correct answer:

a) 2 only
b) 2 and 3 only
c) 1 and 2 only
d) 1, 2 and 3

Q.21) Solution (d)

Article 21 of Constitution of India – “No person shall be deprived of his life or personal liberty
except according to procedure established by law.”

The Indian judiciary has included under the umbrella of Right to life and liberty, Right to free
and fair trial, Right to privacy, Right to clean environment, Right to food, sleep and even
electricity. The courts have not limited the ambit of life to the above rights but also to a
plethora of others.

www.iasbaba.com Page 19
POLITY

The question of interpretation of this paltry worded provision (Article 21) possessing immense
importance came before the Supreme Court as early as in 1953 in the case of A.K. Gopalan v.
State of Madras.

The judiciary has adopted judicial activism to put fetters to governmental actions by
safeguarding public interests through the liberal interpretation of the fundamental rights.

In the case of S. Khushboo v. Kanniammal & Anr., the Supreme Court held that living together is
a right to life under article 21.

Chapter 6: Directive Principles of State Policy

Q.22) According to Dr B R Ambedkar, which among the following is a ‘novel feature’ of the
Indian Constitution?

a) Preamble
b) Judicial Review
c) Fundamental Rights
d) Directive Principles of State Policy

Q.22) Solution (d)

According to Dr B R Ambedkar, the Directive Principles of State Policy is a ‘novel feature’ of the
Indian Constitution.

Dr. Ambedkar said “The directive principles are like instruments of instructions which were
issued to the Governor in General and Governors of colonies and to those of India by the British
Government under the 1935 Act under the Draft Constitution. It is proposed to issue such
instructions to the president and governors. The text of these instruments of the instructions
shall be found in scheduled IV to the Constitution of India. What are called directive principles is
that they are instructions to the Legislature and the Executive. Such a thing is, to my mind, to
be welcomed.”

The Directive Principles commit the State to promote the welfare of the people by affirming
social, economic and political justice, as well as to fight economic inequality.

www.iasbaba.com Page 20
POLITY

Q.23) Consider the following statements regarding Uniform Civil Code:

1. It is a Directive Principle of State Policy given under article 45 of Indian Constitution.


2. It is a Gandhian directive principle.
3. Uniform Civil Code refers to the body of laws governing rights and duties pertaining to
property and personal matters like marriage, divorce, adoption and inheritance.

Which of the above statements are correct?

a) All of the above


b) 2 and 3
c) 1 and 2
d) 3 only

Q.23) Solution (d)

 It is a DPSP given in article-44


 It is a Liberal- Intellectual principle.

Q.24) Consider the following statements about Directive Principles of State Policy:

1. The Directive Principles resemble the ‘Objective Resolutions’ which was moved by
Nehru in 1946.
2. They lay down the foundation stone of social equality and social justice.
3. DPSP consists of certain rights that individuals should enjoy apart from the Fundamental
Rights.

Select the correct answer:

a) 2 only
b) 3 only
c) 1 and 3 only
d) 1, 2 and 3

Q.24) Solution (b)

The Directive Principles resemble the ‘Instrument of Instructions’ enumerated in the


Government of India Act of 1935.

Fundamental Rights lay down the foundation stone of social equality and social justice.

www.iasbaba.com Page 21
POLITY

The chapter on Directive Principles lists mainly three things:


1. The goals and objectives that we as a society should adopt;
2. Certain rights that individuals should enjoy apart from the Fundamental Rights; and
3. Certain policies that the government should adopt.
Q.25) The Directive Principles of State Policy in Indian Constitution have been inspired by the
Directive Principles given in the –

a) Constitution of Ireland
b) Weimar Constitution of Germany
c) Canadian Constitution
d) Constitution of French

Q.25) Solution (a)

The concept of Directive Principles of State Policy was borrowed from the Irish Constitution.
The makers of the Constitution of India were influenced by the Irish nationalist movement,
particularly the Irish Home Rule Movement. Hence, the Directive Principles of the Indian
constitution have been greatly influenced by the Irish Directive Principles of Social Policy.

Do you know?

 Other features borrowed from Irish Constitution are nomination of members to Rajya
Sabha and method of election of president.

Q.26) Which of the below given Directive Principles are part of Gandhian ideology?

1. To make provision for just and humane conditions for work and maternity relief.
2. To promote equal justice and to provide free legal aid to the poor.
3. To prohibit the consumption of intoxicating drinks and drugs which are injurious to
health.
4. To prohibit the slaughter of cows, calves and other milch and draught cattle and to
improve their breeds.

Choose the correct answer:

a) 1 and 3
b) 3 and 4
c) 1, 3 and 4
d) 2, 3 and 4

www.iasbaba.com Page 22
POLITY

Q.26) Solution (b)

Gandhian Principles

In order to fulfil the dreams of Gandhi, some of his ideas were included as Directive Principles.
They require the State:

1. To organise village panchayats and endow them with necessary powers and authority to
enable them to function as units of self-government (Article 40).
2. To promote cottage industries on an individual or co-operation basis in rural areas
(Article 43).
3. To promote voluntary formation, autonomous functioning, democratic control and
professional management of co-operative societies (Article 43B).
4. To promote the educational and economic interests of SCs, STs, and other weaker
sections of the society and to protect them from social injustice and exploitation (Article
46).
5. To prohibit the consumption of intoxicating drinks and drugs which are injurious to
health (Article 47).
6. To prohibit the slaughter of cows, calves and other milch and draught cattle and to
improve their breeds (Article 48).

Directive Principles provided in statements (1) and (2) reflect the ideology of socialism and are
part of Socialistic Principles (not Gandhian Principles).

Q.27) Which among the following Directive Principles were added later and was not part of
the original list?

1. State shall secure opportunities for healthy development of children.


2. State shall take steps to secure the participation of workers in the management of
industries.
3. State shall take steps to protect and improve the environment and to safeguard forests
and wildlife.

Choose correct answer:

a) 3 only
b) 1 only
c) 1 and 3 only
d) 1, 2 and 3

www.iasbaba.com Page 23
POLITY

Q.27) Solution (d)

Four new Directive Principles were added in the 42nd Amendment Act of 1976 to the original
list. They are requiring the state:

1. Added clause in Article 39: To secure opportunities for healthy development of children
2. Added clause in Article 39 as Article 39A: To promote equal justice and to provide free
legal aid to the poor
3. Added clause in Article 43 as Article 43 A: To take steps to secure the participation of
workers in the management of industries
4. Added clause in Article 48 as Article 48A: To protect and improve the environment and
to safeguard forests and wildlife

Source:

 Indian Polity – M Laxmikanth (Chapter 8: Directive Principles of State Policy)


 Introduction to Indian Constitution – D.D.Basu (Chapter: Directive Principles of State
Policy)

Chapter 7: Fundamental Duties

Q.28) Fundamental Duties of India are inspired by which of the following Constitutions?

a) Weimar Constitution of Germany


b) Erstwhile USSR
c) South Africa
d) Ireland

Q.28) Solution (b)

The Fundamental Duties in the Indian Constitution are inspired by the Constitution of erstwhile
USSR. Notably, none of the Constitutions of major democratic countries like USA, Canada,
France, Germany, Australia and so on specifically contain a list of duties of citizens. Japanese
Constitution is, perhaps, the only democratic Constitution in world which contains a list of
duties of citizens. The socialist countries, on the contrary, gave equal importance to the
fundamental rights and duties of their citizens. Hence, the Constitution of erstwhile USSR

www.iasbaba.com Page 24
POLITY

declared that the citizen’s exercise of their rights and freedoms was inseparable from the
performance of their duties and obligations.

Q.29) The provision which says – “to renounce practices derogatory to the dignity of women”
in Indian Constitution is provided in

a) Preamble
b) Fundamental Rights
c) Fundamental Duties
d) DPSPs

Q.29) Solution (c)

Article 51 A (e) provides that - It shall be the duty of every citizen of India to promote harmony
and the spirit of common brotherhood amongst all the people of India transcending religious,
linguistic and regional or sectional diversities; to renounce practices derogatory to the dignity of
women.

Q.30) Consider the following statements about Fundamental Duties.

1. Fundamental duties in Part IV A of constitution are confined only to citizens and not
extended to foreigners.
2. Constitution contains duties of citizens and not the duties of state.

Which of the above statements is/are correct?

a) 1 only
b) 2 only
c) Both 1 and 2
d) None

Q.30) Solution (a)

Unlike some of the Fundamental Rights which extend to all persons whether citizens or
foreigners, the Fundamental Duties are confined to citizens only and do not extend to
foreigners.

www.iasbaba.com Page 25
POLITY

Though the rights and duties of the citizens are correlative and inseparable, the original
constitution contained only the fundamental rights and not the fundamental duties. In other
words, the framers of the Constitution did not feel it necessary to incorporate the fundamental
duties of the citizens in the Constitution. However, they incorporated the duties of the State in
the Constitution in the form of Directive Principles of State Polity. Later in 1976, the
fundamental duties of citizens were added in the Constitution.

Do you know?

 Like the Directive Principles, the fundamental duties are also non-justiciable.

Chapter 8: Union and its Territory

Q.31) Consider the following statements:

1. Process for changing the name of a state or its district/villages can be initiated by state
only.
2. Formation of new states, altering boundary and name is not considered as
Constitutional Amendment under Art 368.
3. Parliament has power to change the name of a state or its districts and villages.

Which of the statements provided above is/are correct?

a) 2 only
b) 2 and 3 only
c) 1 only
d) 1, 2 and 3

Q.31) Solution (a)

Statement (1) is incorrect because Parliament has the power to “change the name of a state”
and it doesn’t matter whether state initiated or not.

Process for changing the name of a state can be initiated by state itself. However, by virtue of
article 3, the parliament has power to change the name of a state even if such proposal does
not come from the concerned state.

www.iasbaba.com Page 26
POLITY

Statement (3) is incorrect because “changing of name of districts/villages involves only the
state legislature and not the Parliament”.

Constitution authorises the Parliament to form new states or alter the areas, boundaries or
names of the existing states without the consent of concerned state legislature or union
territory. In other words, Parliament can redraw the political map of India according to its will.
Hence, the territorial integrity or continued existence of any state is not guaranteed by the
Constitution.

Constitution (Article 4) itself declares that laws made for admission or establishment of new
states (under Article 2) and formation of new states and alteration of areas, boundaries or
names of existing states (under Articles 3) are not to be considered as amendments of the
Constitution under Article 368. This means that such laws can be passed by a simple majority
and by the ordinary legislative process. Hence, Statement (2) is correct.

Do you know?

 The Home Ministry examines the proposal for changing the names of villages, cities and
railway stations once the proposal is received from the State government.
 In other words, Union Home Ministry approves the change of name and conveys the
decision to the State government.
 Between January 2017 and February 2018, Home Ministry has received 27 proposals
from States requesting a change of names of villages, towns and railway stations.

Q.32) Which among the following powers are granted to the Parliament by Article 2 of the
Indian Constitution?

1. power to admit into the Union of India new states


2. power to increase the area of any state
3. power to alter the boundaries of any state
4. power to establish new states which were previously not part of India

Select the correct code:

a) 1 and 2 only
b) 2 and 3 only
c) 1 and 4 only
d) 1, 2, 3 and 4

www.iasbaba.com Page 27
POLITY

Q.32) Solution (c)

Article 2 empowers the Parliament to ‘admit into the Union of India, or establish, new states on
such terms and conditions as it thinks fit’. Thus, Article 2 grants two powers to the Parliament:

(a) the power to admit into the Union of India new states; and
(b) the power to establish new states.

The first refers to the admission of states which are already in existence while the second refers
to the establishment of states which were not in existence before.

Notably, Article 2 relates to the admission or establishment of new states that are not part of
the Union of India.

Article 3, on the other hand, relates to the formation of or chan-ges in the existing states of the
Union of India. In other words, Article 3 deals with the internal re-adjustment inter se of the
territories of the constituent states of the Union of India.

Q.33) Consider the below statements

1. Laws made for admission or establishment of new states and formation of new states
and alteration of areas or boundaries are not considered as amendments of the
Constitution under Article 368.
2. Territorial integrity or continued existence of any state is not guaranteed by the
Constitution.

Which of the statements given above is/are correct?

a) 1 only
b) 2 only
c) Both 1 and 2
d) Nether 1 nor 2

Q.33) Solution (c)

Constitution authorises the Parliament to form new states or alter the areas, boundaries or
names of the existing states without the consent of concerned state legislature or union
territory. In other words, Parliament can redraw the political map of India according to its will.

www.iasbaba.com Page 28
POLITY

Hence, the territorial integrity or continued existence of any state is not guaranteed by the
Constitution.

Do you know?

Constitution (Article 4) itself declares that laws made for admission or establishment of new
states (under Article 2) and formation of new states and alteration of areas, boundaries or
names of existing states (under Articles 3) are not to be considered as amendments of the
Constitution under Article 368. This means that such laws can be passed by a simple majority
and by the ordinary legislative process.

Q.34) Consider the following statements about the Union and territories of India.

1. The territories of India share federal power with the center.


2. The Union of India includes the states only.
3. The constitution describes India as, ‘India that is Bharat, shall be Union of states’.
4. The settlement of border dispute requires constitutional amendment.

Which of the given statements is/are correct?

a) 1 only
b) 2 and 3 only
c) 1 and 3 only
d) None

Q.34) Solution (b)

The union of India includes states which shares federal powers with center. The territory of
India includes the entire territory over which sovereignty is exercised. Apart from states it
includes union territories (which does not share federal power with center) and other
territories acquired by India.

The article 1 describes India, that is, Bharat as a ‘Union of states’.

Supreme Court in 1969 ruled that, settlement of border dispute between India and other
countries does not require constitutional amendment. It can be done by executive action as it
does not involve cession of Indian territory to foreign country.

Do you know?

www.iasbaba.com Page 29
POLITY

 Andhra is the first state in India to be created on linguistic basis, and now the state is
divided on developmental issues into Andhra and Telangana.

Q.35) Arrange the below given states in correct chronological sequence based on their
creation after the reorganisation of the states in 1956:

1. Haryana
2. Nagaland
3. Gujarat
4. Sikkim

Choose the correct code:

a) 4-1-2-3
b) 3-2-1-4
c) 4-2-1-3
d) 3-1-2-4

Q.35) Solution (b)

Gujarat – In 1960, the bilingual state of Bombay was divided into two separate states—
Maharashtra for Marathi-speaking people and Gujarat for Gujarati-speaking people.

Nagaland – In 1963, the State of Nagaland was formed by taking the Naga Hills and Tuensang
area out of the state of Assam.

Haryana – In 1966, the State of Punjab was bifurcated to create Haryana and the union
territory of Chandigarh.

Sikkim – In 1947, after the lapse of British paramountcy, Sikkim became a ‘protectorate’ of
India, whereby the Indian Government assumed responsibility for the defence, external affairs
and communications of Sikkim. In 1974, Sikkim expressed its desire for greater association with
India.

Chapter: 9 Citizenship

www.iasbaba.com Page 30
POLITY

Q.36) The power to grant Indian citizenship according to Citizenship Act lies with the

a) Ministry of Home Affairs


b) Cabinet Secretariat
c) President
d) Prime Minister’s Office

Q.36) Solution (a)

The power to grant citizenship lies only with the Home Ministry under the Citizenship Act.

Q.37) By process of Deprivation, the citizenship of a person can be terminated on which of


the following grounds?

1. When the citizen has obtained the citizenship by fraud


2. The citizen has shown disloyalty towards the constitution of India
3. The citizen has unlawfully traded or communicated with the enemy during a war
4. The citizen has, within five years after registration or naturalisation, been imprisoned in
any country for two years.

Select the code from following:

a) 1, 2 and 3
b) 2, 3 and 4
c) 1, 3 and 4
d) All of the above

Q.37) Solution (d)

The Citizenship Act, 1955, prescribes three ways of losing citizenship whether acquired under
the Act or prior to it under the Constitution, viz, renunciation, termination and deprivation:

1. By Renunciation Any citizen of India of full age and capacity can make a declaration
renouncing his Indian citizenship. Upon the registration of that declaration, that person ceases
to be a citizen of India. However, if such a declaration is made during a war in which India is
engaged, its registration shall be withheld by the Central Government. Further, when a person

www.iasbaba.com Page 31
POLITY

renounces his Indian citizenship, every minor child of that person also loses Indian citizenship.
However, when such a child attains the age of eighteen, he may resume Indian citizenship.

2. By Termination When an Indian citizen voluntarily (consciously, knowingly and without


duress, undue influence or compulsion) acquires the citizenship of another country, his Indian
citizenship automatically terminates. This provision, however, does not apply during a war in
which India is engaged.

3. By Deprivation It is a compulsory termination of Indian citizenship by the Central


government, if:

(a) the citizen has obtained the citizenship by fraud:


(b) the citizen has shown disloyalty to the Constitution of India:
(c) the citizen has unlawfully traded or communicated with the enemy during a war;
(d) the citizen has, within five years after registration or naturalisation, been imprisoned in
any country for two years; and
(e) the citizen has been ordinarily resident out of India for seven years continuously.

Q.38) Under articles 5-8 of the constitution, which of the following persons became citizens of
India at the commencement of the Constitution?

1. Citizenship by descent.
2. Citizenship by registration.
3. Citizenship by naturalization.
4. Citizenship by incorporation of territory.

Select the correct answer using the codes below.

a) 1 only
b) 2 and 3 only
c) 1 and 3 only
d) None of the above

Q.38) Solution (d)

Citizenship at the commencement of the Constitution

 Article 5 -At the commencement of this Constitution, every person who has his domicile
in the territory of India.

www.iasbaba.com Page 32
POLITY

 Article 6 -Rights of citizenship of certain persons who have migrated to India from
Pakistan.
 Article 7 - Rights of citizenship of certain migrants to Pakistan.
 Rights of citizenship of certain persons of Indian origin residing outside India.

Do you know?

 Constitution of India recognises only one domicile, namely, the domicile in India.

Q.39) Through which of the following modes can Indian citizenship be lost?

1. Naturalisation
2. Renunciation
3. Termination
4. Deprivation

Select the code from following:

a) 1,2 and 3
b) 2,3 and 4
c) 1,3 and 4
d) All of the above

Q.39) Solution (b)

The Citizenship Act, 1955, prescribes three ways of losing citizenship whether acquired under
the Act or prior to it under the Constitution, viz, renunciation, termination and deprivation:

1. By Renunciation Any citizen of India of full age and capacity can make a declaration
renouncing his Indian citizenship. Upon the registration of that declaration, that person ceases
to be a citizen of India. However, if such a declaration is made during a war in which India is
engaged, its registration shall be withheld by the Central Government.
Further, when a person renounces his Indian citizenship, every minor child of that person also
loses Indian citizenship. However, when such a child attains the age of eighteen, he may
resume Indian citizenship.

2. By Termination When an Indian citizen voluntarily (consciously, knowingly and without


duress, undue influence or compulsion) acquires the citizenship of another country, his Indian

www.iasbaba.com Page 33
POLITY

citizenship automatically terminates. This provision, however, does not apply during a war in
which India is engaged.

3. By Deprivation It is a compulsory termination of Indian citizenship by the Central


government, if:
(a) the citizen has obtained the citizenship by fraud:
(b) the citizen has shown disloyalty to the Constitution of India:
(c) the citizen has unlawfully traded or communicated with the enemy during a war;
(d) the citizen has, within five years after registration or naturalisation, been imprisoned in
any country for two years; and
(e) the citizen has been ordinarily resident out of India for seven years continuously.

Note: Naturalisation is the process of attaining the Indian Citizenship.

Q.40) If any foreign territory becomes a part of India –

a) Government of India specifies the persons who among the people of the territory shall
be the citizens of India.
b) Automatically all the people belonging to the territory become the citizens of India.
c) Parliament has to amend the Citizenship Act and make provision for providing
citizenship to people of the territory.
d) People of the territory can acquire citizenship of India by Registration.

Q.40) Solution (a)

If any foreign territory becomes a part of India, the Government of India specifies the persons
who among the people of the territory shall be the citizens of India. Such persons become the
citizens of India from the notified date. For example, when Pondicherry became a part of India,
the Government of India issued the Citizenship (Pondicherry) Order, 1962, under the
Citizenship Act, 1955.

Chapter 10. Amendment of the Constitution

www.iasbaba.com Page 34
POLITY

Q.41) Article 368 of Indian Constitution provides a provision for the amendment of the
Constitution. What are different ways in which different parts of Indian Constitution can be
amended?

1. Amendment by simple majority of the Parliament


2. Amendment by special majority of the Parliament
3. Amendment by special majority of the Parliament and the ratification of half of the state
legislatures.

Select the code from below:

a) 2 and 3
b) 1 and 2
c) 1 and 3
d) All of the above

Q.41) Solution (d)

Article 368 provides for two types of amendments, that is, by a special majority of Parliament
and also through the ratification of half of the states by a simple majority. But, some other
articles provide for the amendment of certain provisions of the Constitution by a simple
majority of Parliament, that is, a majority of the members of each House present and voting
(similar to the ordinary legislative process). Notably, these amendments are not deemed to be
amendments of the Constitution for the purposes of Article 368.

Therefore, the Constitution can be amended in three ways:

(a) Amendment by simple majority of the Parliament,

(b) Amendment by special majority of the Parliament, and

(c) Amendment by special majority of the Parliament and the ratification of half of the state
legislatures.

Q.42) Which of the following are matters on which a constitutional amendment is possible
only with the ratification of the legislatures of not less than one-half of the States?

1. Election of the President


2. Representation of States in Parliament

www.iasbaba.com Page 35
POLITY

3. Any of the Lists in the 7th Schedule


4. Abolition of the Legislative Council of a State

Choose the correct answer from the codes given below:

a) 1, 2 and 3
b) 1, 2 and 4
c) 1, 3 and 4
d) 2, 3 and 4

Q.42) Solution (a)

Article 169 in the Constitution of India 1949: Abolition or creation of Legislative Councils in
States.

Parliament may by law provide for the abolition of the Legislative Council of a State having such
a Council or for the creation of such a Council in a State having no such Council, if the
Legislative Assembly of the State passes a resolution to that effect by a majority of the total
membership of the Assembly and by a majority of not less than two thirds of the members of
the Assembly present and voting.

Q.43) Consider the following statements regarding Constitution Amendment Bill:

1. It cannot be initiated in State Legislatures.


2. It cannot be initiated by a Private member in Parliament.

Which of the above statements are correct?

a) 1 only
b) 2 only
c) Both 1 and 2
d) Neither 1 nor 2

Q.43) Solution (a)

Procedure of Constitution Amendment

As per the procedure laid out by article 368 for amendment of the Constitution, an amendment
can be initiated only by the introduction of a Bill in either House of Parliament. The Bill must
then be passed in each House by a majority of the total membership of that House and by a

www.iasbaba.com Page 36
POLITY

majority of not less than two-thirds of the members of that House present and voting. There is
no provision for a joint sitting in case of disagreement between the two Houses. Total
membership in this context has been defined to mean the total number of members comprising
the House irrespective of any vacancies or absentees on any account vide Explanation to Rule
159 of the Rules of Procedure and Conduct of Business in Lok Sabha.

The Bill, passed by the required majority, is then presented to the President who shall give his
assent to the Bill. If the amendment seeks to make any change in any of the provisions
mentioned in the proviso to article 368, it must be ratified by the Legislatures of not less than
one-half of the States. These provisions relate to certain matters concerning the federal
structure or of common interest to both the Union and the States viz., the election of the
President (articles 54 and 55); the extent of the executive power of the Union and the States
(articles 73 and 162); the High Courts for Union territories (article 241); The Union Judiciary and
the High Courts in the States (Chapter IV of Part V and Chapter V of Part VI); the distribution of
legislative powers between the Union and the States (Chapter I of Part XI and Seventh
Schedule); the representation of States in Parliament; and the provision for amendment of the
Constitution laid down in article 368. Ratification is done by a resolution passed by the State
Legislatures. There is no specific time limit for the ratification of an amending Bill by the State
Legislatures. However, the resolutions ratifying the proposed amendment must be passed
before the amending Bill is presented to the President for his assent.

Note: Any member of the House can initiate the Constitution Amendment Bill. It can be done
only in the Parliament and not in State Legislature.

Q.44) Which of the following amendments are correctly matched?

1. 89th Amendment Act - The National Commission for SC and ST was bifurcated
2. 61st Amendment Act – Reduce age of voting from 21 years to 18 years
3. 92nd Amendment Act – Changed anti-defection laws
4. 69th Amendment Act – Establishment of legislative assembly and council of ministers
for Federal National Capital of Delhi

Select the code from following:

a) 1,2 and 3
b) 2,3 and 4
c) 1,2 and 4
d) All of the above

www.iasbaba.com Page 37
POLITY

Q.44) Solution (c)

91st Constitution Amendment Act, 2003

91st Amendment Act - Restrict the size of council of ministers to 15% of legislative members &
to strengthen Anti Defection laws

Prior to January 1, 2004 (effective date of 91st Amendment of the Constitution) the Prime
Minister had discretion to appoint any number in his council of ministers. But the Constitution
(Ninety-first Amendment) Act in 2003 made a drastic change in curbing such power of the
Prime Minister.

This Amendment added clause (1A) in this Article which made a specific provision that, the total
number of Ministers, including Prime Minister, in no case can exceed 15 per cent of the total
number of Lok Sabha members.

The Prime Minister can induct into his ministry a person who is not a member of either House
of Parliament. However, a minister who for a period of six consecutive months is not a member
of either House of Parliament shall at the expiration of that period cease to be one.

Chapter 11: Basic Structure of the Constitution

Q.45) Supreme court in Kesavananda Bharati case (1973) laid down a new doctrine of the
‘basic structure’ (‘basic features’) of the Constitution. Consider the following statements
about basic structure of constitution.
1. The Supreme Court in the Minerva Mills case (1980) defined what constitutes basic
structure.
2. Welfare state is a component of basic structure.

Which of the above statements is/are correct?


a) 1 only
b) 2 only
c) Both 1 and 2
d) None

Q.45) Solution (b)


The Supreme Court is yet to define or clarify as to what constitutes the ‘basic structure’ of the
Constitution. From the various judgements, the following have emerged as ‘basic features’ of

www.iasbaba.com Page 38
POLITY

the Constitution or elements / components / ingredients of the ‘basic structure’ of the


constitution:

 Supremacy of the Constitution


 Sovereign, democratic and republican nature of the Indian polity
 Secular character of the Constitution
 Separation of powers between the legislature, the executive and the judiciary
 Federal character of the Constitution
 Unity and integrity of the nation
 Welfare state (socio-economic justice)
 Judicial review
 Freedom and dignity of the individual
 Parliamentary system
 Rule of law
 Harmony and balance between Fundamental Rights and Directive
Principles
 Principle of equality
 Free and fair elections
 Independence of Judiciary
 Limited power of Parliament to amend the Constitution
 Effective access to justice
 Principles (or essence) underlying fundamental rights.
 Powers of the Supreme Court under Articles 32, 136, 141 and 142
 Powers of the High Courts under Articles 226 and 227

Do you know?

 In Indira Nehru Gandhi case (1975) (popularly known as the Election Case) supreme
court declared that `Government of laws and not of men’ (i.e., Rule of Law) as basic
structure of constitution.

Chapter 12: Parliamentary System

Chapter 13: Federal System

www.iasbaba.com Page 39
POLITY

Q.46) In India the Parliament is not supreme (sovereign body) and enjoys
limited and restricted powers. Which of the following features are responsible for this
limitation on Parliament?
1. Written Constitution.
2. Federal system.
3. Judicial review.
4. Fundamental rights.

Select the correct answer using the codes given below.

a) 1 and 2 only
b) 1 and 3 only
c) 3 and 4 only
d) All the above

Q.56) Solution (d)

The doctrine of ‘sovereignty of Parliament’ is associated with the British


Parliament. Sovereignty means the supreme power within the State. That supreme power in
Great Britain lies with the Parliament. There are no ‘legal’ restrictions on its authority and
jurisdiction.
The Indian Parliament, on the other hand, cannot be regarded as a
sovereign body in the similar sense as there are ‘legal’ restrictions on its
authority and jurisdiction. The factors that limit the sovereignty of Indian
Parliament are: written Constitution, federal system, judicial review and fundamental rights.
Do you know?

 ‘Shadow cabinet’ is a unique institution of the British cabinet system. It is formed by the
opposition party to balance the ruling cabinet and to prepare its members for future
ministerial office. There is no such institution in India.

Q.47) In which of the following case, the Supreme Court laid down that the Constitution is
federal and characterised federalism as its ‘basic feature’?

a) Bommai case (1994)


b) Minerva Mills case (1980)
c) Cooper case (1970)
d) Shankari Prasad (1951)

www.iasbaba.com Page 40
POLITY

Q.47) Solution (a)

In a landmark judgement in Bommai case (1994), the Supreme Court laid down that the
Constitution is federal and characterised federalism as its ‘basic feature’. Therefore, Option (a)
is correct.

Chaper 14: Centre–State Relations

Chapter 15: Inter-State Relations

Q.48) The Constitution empowers the Parliament to legislate on any matter enumerated in
State List under which of the following circumstances?

1. When Rajya Sabha Passes a resolution for the same


2. During a National Emergency
3. When legislature of a state passes a resolution and request the parliament to make law
for that state on some specific subject
4. The Parliament can make laws on any matter in the State List for implementing the
international treaties, agreements or conventions

Select the code from below:

a) 1, 2 and 3
b) 2, 3 and 4
c) 1, 2 and 4
d) All of the above

Q.48) Solution (c)

The Constitution empowers the Parliament to make laws on any matter enumerated in the
State List under the following five extraordinary circumstances:

1. When Rajya Sabha Passes a Resolution

 If the Rajya Sabha declares that it is necessary in the national interest that Parliament
should make laws on a matter in the State List, then the Parliament becomes competent

www.iasbaba.com Page 41
POLITY

to make laws on that matter. Such a resolution must be supported by two-thirds of the
members present and voting. The resolution remains in force for one year; it can be
renewed any number of times but not exceeding one year at a time. The laws cease to
have effect on the expiration of six months after the resolution has ceased to be in
force.
 This provision does not restrict the power of a state legislature to make laws on the
same matter. But, in case of inconsistency between a state law and a parliamentary law,
the latter is to prevail.

2. During a National Emergency

 The Parliament acquires the power to legislate with respect to matters in the State List,
while a proclamation of national emergency is in operation. The laws become
inoperative on the expiration of six months after the emergency has ceased to operate.
 Here also, the power of a state legislature to make laws on the same matter is not
restricted. But, in case of repugnancy between a state law and a parliamentary law, the
latter is to prevail.

3. When States Make a Request

 When the legislatures of two or more states pass resolutions requesting the Parliament
to enact laws on a matter in the State List, then the Parliament can make laws for
regulating that matter. A law so enacted applies only to those states which have passed
the resolutions. However, any other state may adopt it afterwards by passing a
resolution to that effect in its legislature. Such a law can be amended or repealed only
by the Parliament and not by the legislatures of the concerned states.
 The effect of passing a resolution under the above provision is that the Parliament
becomes entitled to legislate with respect to a matter for which it has no power to make
a law. On the other hand, the state legislature ceases to have the power to make a law
with respect to that matter. The resolution operates as abdication or surrender of the
power of the state legislature with respect to that matter and it is placed entirely in the
hands of Parliament which alone can then legislate with respect to it.
 Some examples of laws passed under the above provision are Prize Competition Act,
1955; WildLife (Protection) Act, 1972; Water (Prevention and Control of Pollution) Act,
1974; Urban Land (Ceiling and Regulation) Act, 1976; and Transplantation of Human
Organs Act, 1994.

4. To Implement International Agreements

www.iasbaba.com Page 42
POLITY

 The Parliament can make laws on any matter in the State List for implementing the
international treaties, agreements or conventions. This provision enables the Central
government to fulfill its international obligations and commitments.
 Some examples of laws enacted under the above provision are United Nations
(Privileges and Immunities) Act, 1947; Geneva Convention Act, 1960; Anti-Hijacking Act,
1982 and legislations relating to environment and TRIPS.

5. During President’s Rule

 When the President’s rule is imposed in a state, the Parliament becomes empowered to
make laws with respect to any matter in the State List in relation to that state. A law
made so by the Parliament continues to be operative even after the president’s rule.
This means that the period for which such a law remains in force is not co-terminus with
the duration of the President’s rule. But, such a law can be repealed or altered or re-
enacted by the state legislature.

Q.49) Which of the following statements are incorrect for Statutory Grants?

a) Statutory Grants are a type of Grants – in – Aid given to States by the Center.
b) They are provided to states which are in need of financial assistance and not to every
state.
c) These sums are charged on the Consolidated fund of India.
d) They were given on the recommendation of Planning Commission, and now will be
come under NITI Aayog.

Q.49) Solution (d)

Grants-in-Aid to the States

 Besides sharing of taxes between the Centre and the states, the Constitution provides
for grants-in-aid to the states from the Central resources. There are two types of grants-
in-aid, viz, statutory grants and discretionary grants:

Statutory Grants

 Article 275 empowers the Parliament to make grants to the states which are in need of
financial assistance and not to every state. Also, different sums may be fixed for
different states. These sums are charged on the Consolidated Fund of India every year.

www.iasbaba.com Page 43
POLITY

 Apart from this general provision, the Constitution also provides for specific grants for
promoting the welfare of the scheduled tribes in a state or for raising the level of
administration of the scheduled areas in a state including the State of Assam.
 The statutory grants under Article 275 (both general and specific) are given to the states
on the recommendation of the Finance Commission.

Discretionary Grants

 Article 282 empowers both the Centre and the states to make any grants for any public
purpose, even if it is not within their respective legislative competence.

Q.50) The objectives (or the functions) of the zonal councils does not include which of the
following:

a) To help in arresting the growth of acute state-consciousness, regionalism, linguism and


particularistic trends
b) To help in removing the after-effects of separation in some cases so that the process of
reorganisation, integration and economic advancement may synchronise
c) To secure some kind of political equilibrium between different regions of the country
d) None of the above

Q.50) Solution (d)

The objectives (or the functions) of the zonal councils, in detail, are as follows:
 To achieve an emotional integration of the country.
 To help in arresting the growth of acute state-consciousness, regionalism, linguism and
particularistic trends.
 To help in removing the after-effects of separation in some cases so that the process of
reorganisation, integration and economic advancement may synchronise.
 To enable the Centre and states to cooperate with each other in social and economic
matters and exchange ideas and experience in order to evolve uniform policies.
 To cooperate with each other in the successful and speedy execution of major
development projects.
 To secure some kind of political equilibrium between different regions of the country.

Chapter 16: Emergency Provisions

www.iasbaba.com Page 44
POLITY

Q.51) Under Article 352, the President can declare a national emergency when the security of
India or a part of it is threatened by

1. War
2. External aggression
3. Internal disturbance

Select the correct code:

a) 1 only
b) 2 only
c) 1 and 2 only
d) All of the above

Q.51) Solution (c)

Explanation:
 Under Article 352, the President can declare a national emergency when the security of
India or a part of it is threatened by war or external aggression or armed rebellion (but not
on the ground of ‘internal disturbance’)

Q.52) By which of the following respect/s, a resolution of disapproval is different from a


resolution approving the continuation of a proclamation of emergency?

1. The first one is required to be passed by the Lok Sabha only, while the second one needs
to be passed by the both Houses of Parliament.
2. The first one is to be adopted by a simple majority only, while the second one needs to
be adopted by a special majority.

Choose the correct codes from below options:

a) 1 only
b) 2 only
c) Both 1 and 2
d) Neither 1 nor 2

Q.52) Solution (c)

Both the statements are correct.

www.iasbaba.com Page 45
POLITY

A resolution of disapproval is different from a resolution approving the continuation of a


proclamation of emergency in the following two respects:
1. The first one is required to be passed by the Lok Sabha only, while the second one needs
to be passed by the both Houses of Parliament.
2. The first one is to be adopted by a simple majority only, while the second one needs to
be adopted by a special majority.

Q.53) Consider the following statements with regard to Parliamentary Approval of the
proclamation of Constitutional Emergency:

1. Once the President proclaims the Constitutional Emergency, it must be approved by


both the Houses of Parliament within one month from the date of its issue.
2. However, if the proclamation of the Constitutional Emergency is issued at a time when
the LS has been dissolved or the dissolution of the LS takes place during the period of
one month without approving the proclamation, then the proclamation survives until 30
days from the first sitting of the Lok Sabha after its reconstitution, provided the Rajya
Sabha has in the meantime approved it.

Which of the statements given above is/are correct?

a) 1 only
b) 2 only
c) Both 1 and 2
d) Neither 1 nor 2

Q.53) Solution (d)

Explanation:
 An Emergency due to the failure of the constitutional machinery in the states (Article 356).
This is popularly known as ‘President’s Rule’. It is also known by two other names—‘State
Emergency’ or ‘Constitutional Emergency’.
 A proclamation imposing President’s Rule must be approved by both the Houses of
Parliament within two months from the date of its issue.
 However, if the proclamation of President’s Rule is issued at a time when the Lok Sabha has
been dissolved or the dissolution of the Lok Sabha takes place during the period of two
months without approving the proclamation, then the proclamation survives until 30 days
from the first sitting of the Lok Sabha after its reconstitution, provided the Rajya Sabha
approves it in the mean time.

www.iasbaba.com Page 46
POLITY

*Note: When it is regard to proclamation of National Emergency, then the given statements
will be correct (i.e. approval by 1 month)

Q.54) Which of the following is not true in regard to the proclamation of Emergency?

a) The proclamation of Emergency must be approved by both the Houses of Parliament


within two months from the date of its issue
b) Every resolution approving the proclamation of emergency or its continuance must be
passed by either House of Parliament by a special majority
c) A proclamation of emergency may be revoked by the President at any time by a
subsequent proclamation. Such a proclamation does not require the parliamentary
approval
d) Further, the President must revoke a proclamation if the Lok Sabha passes a resolution
disapproving its continuation

Q.54) Solution (a)

Explanation:
 The proclamation of Emergency must be approved by both the Houses of Parliament within
one month from the date of its issue

Q.55) Consider the following statements in regard to the National Emergency:

1. During a national emergency, the executive power of the Centre extends to directing
any state regarding the manner in which its executive power is to be exercised.
2. Thus, the state governments are suspended and are brought under the complete
control of the Centre.

Which of the statements given above is/are correct?

a) 1 only
b) 2 only
c) Both
d) None

Q.55) Solution (a)

Explanation:
 During a national emergency, the executive power of the Centre extends to directing any
state regarding the manner in which its executive power is to be exercised.

www.iasbaba.com Page 47
POLITY

 Thus, the state governments are brought under the complete control of the Centre, though
they are not suspended.

Chapter: President

Q.56) President in India is not directly elected. The electoral college for Presidential election
consists of:

1. All the members of Parliament


2. All the members of Legislative Assemblies
3. Elected members of Legislative Assemblies of Delhi and Puducherry

Which of the above is/are correct?

a) 3 only
b) 1 and 2
c) 2 and 3
d) All of the above

Q.56) Solution (a)

The President is elected not directly by the people but by members of electoral college
consisting of:
1. the elected members of both the Houses of Parliament;
2. the elected members of the legislative assemblies of the states; and
3. the elected members of the legislative assemblies of the Union Territories of Delhi and
Puducherry.

Thus, the nominated members of both of Houses of Parliament, the nominated members of the
state legislative assemblies, the members (both elected and nominated) of the state legislative
councils (in case of the bicameral legislature) and the nominated members of the Legislative
Assemblies of Delhi and Puducherry do not participate in the election of the President. Where
an assembly is dissolved, the members cease to be qualified to vote in presidential election,
even if fresh elections to the dissolved assembly are not held before the presidential election.

Q.57) Who among the following takes the oath to ‘uphold the constitution and the laws’?

1. President
2. CAG

www.iasbaba.com Page 48
POLITY

3. Supreme court Judge

Select the correct answer using the codes given below.

a) 1 and 2 only
b) 2 only
c) 3 only
d) 2 and 3 only

Q.57) Solution (d)

Only CAG and a Supreme Court Judge takes the oath to uphold the constitution and the laws.
President takes an oath to preserve, protect and defend the constitution.

Q.58) Constitution lays down which of the following bills to be introduced in the Parliament
only on the recommendation of the President?

a) A bill which imposes or varies any tax or duty in which states are interested
b) A bill which varies the meaning of the expression ‘agricultural income’ as defined for the
purposes of the enactments relating to Indian income tax
c) Both (a) and (b)
d) None of the above

Q.58) Solution (c)

To protect the interest of states in the financial matters, the Constitution lays down that the
following bills can be introduced in the Parliament only on the recommendation of the
President:
 A bill which imposes or varies any tax or duty in which states are interested;
 A bill which varies the meaning of the expression ‘agricultural income’ as defined for the
purposes of the enactments relating to Indian income tax;
 A bill which affects the principles on which moneys are or may be distributable to states;
and
 A bill which imposes any surcharge on any specified tax or duty for the purpose of the
Centre.

www.iasbaba.com Page 49
POLITY

Q.59) The President can be removed from office by a process of impeachment for ‘violation of
the Constitution’. Consider the following statements.
1. Constitution does not define the meaning of the phrase ‘violation of the Constitution’.
2. Constitution does not provide the procedure for impeachment of president and left the
Parliament to frame a law.
3. Parliament enacted President Act 1950 for the impeachment of President.

Which of the above statements is/are correct?


a) 1 only
b) 2 only
c) 2 and 3 only
d) All the above

Q.59) Solution (a)

The President can be removed from office by a process of impeachment for ‘violation of the
Constitution’. However, the Constitution does not define the meaning of the phrase ‘violation
of the Constitution’.
Since Constitution provides the mode and ground for removing the President, he cannot be
removed otherwise than by impeachment, in accordance with the terms of articles 56 and 61.
Do you know?

 The nominated members of either House of Parliament can participate in the


impeachment of the President though they do not participate in his election; the
nominated members of either House of Parliament can participate in the impeachment
of the President though they do not participate in his election;

Q.60) Which of the following amendments affected the executive authority of the President?
1. 42nd Constitutional Amendment Act.
2. 44th Constitutional Amendment Act.
3. 91st Constitutional Amendment Act.
4. 86th Constitutional Amendment Act.

Choose the correct answer using the codes given below.


a) 1 and 2 only
b) 1 and 3 only

www.iasbaba.com Page 50
POLITY

c) 3 only
d) All the above

Q.60) Solution (a)


Article 74 was amended by the 42nd Constitutional Amendment Act of 1976 to the effect that
the President shall, in the exercise of his functions, act in accordance with the advice rendered
by the council of ministers.
The 44th Constitutional Amendment Act of 1978 further added a proviso to this(74th) article
to the effect that the President may require the council of ministers to reconsider such advice
and the president shall act in accordance with the advice tendered after such reconsideration.
The 91st Amendment to the Constitution is related to, limiting the size of the Council of
Ministers at the Centre and the States to no more than 15 per cent of the numbers in the Lok
Sabha or the State Legislature respectively.
Spurred by the Unnikrishnan judgment and a public demand to enforce the right to education,
successive governments from 1993 worked towards bringing a constitutional amendment to
make education a fundamental right. That led to the 86th amendment in December 2002.
Do you know?

 The Supreme court held that ‘wherever the Constitution requires the satisfaction of the
President, the satisfaction is not the personal satisfaction of the President, but it is the
satisfaction of the council of ministers with whose aid and on whose advice the
President exercises his powers and functions’.

Chapter 18 Vice-President

Q.61) Which of the following are correct eligibility qualifications for a person to contest in
Vice – Presidential Election?

1. He should be a citizen of India


2. He should have completed 30 years of age
3. He should be qualified for election as a member of Lok Sabha
4. He should not hold an office of profit under any government authority.

Select the code from below:

a) 1, 2 and 3

www.iasbaba.com Page 51
POLITY

b) 1, 2 and 4
c) 1 and 4
d) All of the above

Q.61) Solution (c)

2nd and 3rd statements are incorrect.

To be eligible for election as Vice-President, a person should fulfil the following qualifications:

 He should be a citizen of India.


 He should have completed 35 years of age.
 He should be qualified for election as a member of the Rajya Sabha.
 He should not hold any office of profit under the Union government or any state
government or any local authority or any other public authority.

Q.62) Recently, there was news that Tamil Nadu government decides to remit the life
sentences of all the seven convicts in the Rajiv Gandhi assassination case. Which among the
following statements is/are true?

1. State has to consult the Centre before releasing prisoners prosecuted by the CBI or
under a Central law
2. Governor of a State has the power to suspend, remit or commute the sentence of any
person convicted of any offence against any law relating to a matter to which the
executive power of the State extends

Choose the appropriate code:

a) 1 only
b) 2 only
c) 1 and 2 only
d) None of the above

Q.62) Solution (c)

Explanation:
 Tamil Nadu government decided to remit the life sentences of all the seven convicts in the
Rajiv Gandhi assassination case and release them as they had already served over 24 years
in prison.

www.iasbaba.com Page 52
POLITY

 Under Section 435 of the Cr.PC, the State has to consult the Centre before releasing
prisoners prosecuted by the CBI or under a Central law.
 In December last year, the Supreme Court had ruled that the State government had no
power to release the Rajiv Gandhi case convicts without the Centre’s concurrence.
 Article 161: Governor of a State has the power to grant pardons, reprieves, respites or
remissions of punishment or to suspend, remit or commute the sentence of any person
convicted of any offence against any law relating to a matter to which the executive power
of the State extends. Please note that President can grant pardon to a person awarded
death sentence. But Governor of State does not enjoy this power.

Q.63) Consider the following statements:

1. Central Legislation can over-ride the veto power of President in the case of suspensive
veto
2. State Legislature cannot over-ride the veto power of President in the case of suspensive
veto

Which of the statements given above is/are correct?

a) 1 only
b) 2 only
c) 1 and 2 only
d) None of the above

Q.63) Solution (c)

Explanation:
Suspensive Veto:
 The President exercises this veto when he returns a bill for reconsideration of the
Parliament. However, if the bill is passed again by the Parliament with or without
amendments and again presented to the President, it is obligatory for the President to give
his assent to the bill. This means that the presidential veto is overridden by a re-passage of
the bill by the same ordinary majority.
 When a bill is reserved by the governor for the consideration of the President, the President
may direct the governor to return the bill (if it is not a money bill) for the reconsideration of
the state legislature. If the bill is passed again by the state legislature with or without
amendments and presented again to the President for his assent, the President is not
bound to give his assent to the bill. This means that the state legislature cannot override the
veto power of the President.

www.iasbaba.com Page 53
POLITY

Q.64) Consider the following with regard to Ordinance making power of President:

1. Ordinance making is the discretionary power of the President of India


2. President can promulgate or withdraw an ordinance only on the advice of Cabinet
Ministers

Which of the statements given above is/are incorrect?

a) 1 only
b) 2 only
c) Both 1 and 2
d) Neither 1 nor 2

Q.64) Solution (c)

Explanation:
 Article 123 of the Constitution empowers the President to promulgate ordinances during
the recess of Parliament. These ordinances have the same force and effect as an act of
Parliament, but are in the nature of temporary laws.
 He can promulgate an ordinance only when both the Houses of Parliament are not in
session or when either of the two Houses of Parliament is not in session. An ordinance can
also be issued when only one House is in session because a law can be passed by both the
Houses and not by one House alone. An ordinance made when both the Houses are in
session is void. Thus, the power of the President to legislate by ordinance is not a parallel
power of legislation.
 However, his power of ordinance-making is not a discretionary power, and he can
promulgate or withdraw an ordinance only on the advice of the council of ministers headed
by the prime minister. (not Cabinet Ministers)

Q.65) Which of the following statements is/are correct with regard to President and Prime
Minister?

1. The President is the real executive while the Prime Minister is the nominal executive
2. The President is head of the State, while the Prime Minister is head of the government
3. There shall be council of ministers headed by the Prime Minister to aid and advise the
President in the exercise of his functions. The advice so tendered is not binding on the
President.

Choose the correct codes from below options:

www.iasbaba.com Page 54
POLITY

a) 1 and 2
b) 2 and 3
c) 2 only
d) All of the above

Q.65) Solution (c)

Explanation:
 The President is the nominal executive (de jure executive or titular executive) while the
Prime Minister is the real executive (de facto executive). Thus, the President is head of
the State, while the Prime Minister is head of the government.
 Article 74 provides for a council of ministers headed by the Prime Minister to aid and
advise the President in the exercise of his functions. The advice so tendered is binding
on the President.

Chapter 19: Governor

Q.66) Which one among the following Constitutional Amendments facilitated the
appointment of the same person as a governor for two or more states?

a) 1st Constitutional Amendment Act, 1951


b) 4th Constitutional Amendment Act, 1955
c) 7th Constitutional Amendment Act, 1956
d) 10th Constitutional Amendment Act, 1961

Q.66) Solution (c)

7th Constitutional Amendment Act of 1956 facilitated the appointment of the same person as a
governor for two or more states.

Q.67) Which one of the following statements is correct?

a) In India, the same person cannot be appointed as governor for two or more states at the
same time.
b) The judges of the high court of the states in India are appointed by the Governor of the
state just as the judges of the Supreme Court are appointed by the president.
c) No procedure has been laid down in the constitution of India for the selection and
appointment of the Chief Minister.

www.iasbaba.com Page 55
POLITY

d) In the case of a union territory having a legislative setup, the chief minister is appointed
by the Lt. Governor on the basis of majority support.

Q.67) Solution (c)

In India, the same person can be appointed as governor for two or more states at the same
time.

The judges of the high court of the states in India are appointed by the President only, just as
the judges of the Supreme Court are appointed by the President.

The Constitution does not contain any specific procedure for the selection and appointment of
the Chief Minister.

Article 164 only says that the Chief Minister shall be appointed by the governor. However, this
does not imply that the governor is free to appoint any one as the Chief Minister.

 In accordance with the convections of the parliamentary system of government, the


governor has to appoint the leader of the majority party in the state legislative assembly
as the Chief Minister.
 But, when no party has a clear majority in the assembly, then the governor may exercise
his personal discretion in the selection and appointment of the Chief Minister.
 In such a situation, the governor usually appoints the leader of the largest party or
coalition in the assembly as the Chief Minister and ask him to seek a vote of confidence
in the House within a month.

Q.68) Consider the following statements below

1. The President can make regulations for the peace, progress and good government of the
four Union Territories—the Andaman and Nicobar Islands, Lakshadweep, Dadra and
Nagar Haveli and Daman and Diu.
2. The governor is empowered to direct that an act of Parliament does not apply to a
scheduled area in the state or apply with specified modifications and exceptions. (i.e.,
Schedule V areas)
3. The President enjoys the same power with respect to tribal areas (autonomous districts)
in Assam, Meghalaya, Tripura and Mizoram. (i.e., Schedule VI areas)

Choose the correct codes from below options:

a) 1 and 2
b) 2 and 3
c) 2 only

www.iasbaba.com Page 56
POLITY

d) All of the above

Q.68) Solution (a)

Explanation:
 The President can make regulations for the peace, progress and good government of the
four Union Territories—the Anda-man and Nicobar Islands, Lakshadweep, Dadra and
Nagar Haveli and Daman and Diu. A regulation so made has the same force and effect as
an act of Parliament. It may also repeal or amend any act of Parliament in relation to
these union territories.
 The governor is empowered to direct that an act of Parliament does not apply to a
scheduled area in the state or apply with specified modifications and exceptions.
 The Governor of Assam may likewise direct that an act of Parliament does not apply to a
tribal area (autonomous district) in the state or apply with specified modifications and
exceptions. The President enjoys the same power with respect to tribal areas
(autonomous districts) in Meghalaya, Tripura and Mizoram.

Chapter 20: Prime Minister

Q.69) Consider the following statements:

1. Article 75 contains the exact procedure for selection and appointment of Prime
Minister.
2. President is the nominal executive authority and Prime Minister is the real executive
authority.
3. Prime Minister has to be a member of Lok Sabha only.

Which of the above statements are incorrect?

a) 1 and 2
b) 2 and 3
c) 1 and 3
d) None of the above

Q.69) Solution (c)

In the scheme of parliamentary system of government provided by the constitution, the


President is the nominal executive authority (de jure executive) and Prime Minister is the real
executive authority (de facto executive). In other words, president is the head of the State
while Prime Minister is the head of the government.

www.iasbaba.com Page 57
POLITY

The Constitution does not contain any specific procedure for the selection and appointment of
the Prime Minister. Article 75 says only that the Prime Minister shall be appointed by the
president.

However, this does not imply that the president is free to appoint any one as the Prime
Minister. In accordance with the conventions of the parliamentary system of government, the
President has to appoint the leader of the majority party in the Lok Sabha as the Prime
Minister.

Constitutionally, the Prime Minister may be a member of any of the two Houses of parliament.
For example, three Prime Ministers, Indira Gandhi (1966), Deve Gowda (1996) and Manmohan
Singh (2004), were members of the Rajya Sabha.

www.iasbaba.com Page 58
POLITY

Chapter 21. Central Council of Ministers

Chapter 22. Cabinet Committees

Q.70) Which of the following statements regarding Council of Ministers are correct?

1. The Advice of Council of Ministers is binding on the President.


2. The nature of advice tendered by the Ministers can be questioned in the Supreme
Court.

Select the code from below:

a) 1 only
b) 2 only
c) Both 1 and 2
d) Neither 1 nor 2

Q.70) Solution (a)

Article 74 provides for a council of ministers with the Prime Minister at the head to aid and
advise the President in the exercise of his functions. The 42nd and 44th Constitutional
Amendment Acts have made the advice binding on the President.1 Further, the nature of
advice tendered by ministers to the President cannot be enquired by any court. This provision
emphasises the intimate and the confidential relationship between the President and the
ministers.

Q.71) Which among the following is/are correct statements in regard to Council of ministers
and Cabinet?

1. Cabinet functions are determined by the council of ministers.


2. Council of ministers implements the decisions taken by the cabinet.

Choose the appropriate code:

a) 1 only
b) 2 only
c) Both 1 and 2

www.iasbaba.com Page 59
POLITY

d) Neither 1 nor 2

Q.71) Solution (b)

Council of ministers functions are determined by the cabinet.

Council of ministers implements the decisions taken by the cabinet. Cabinet supervises the
implementation of its decisions by the council of ministers.

Q.72) Consider the following statements:

1. The Council of minister is responsible to Lok Sabha.


2. Council of ministers can be removed by passing of Vote of No Confidence in Rajya
Sabha.

Which of the above statements are incorrect?

a) 1 only
b) 2 only
c) Both 1 and 2
d) Neither 1 nor 2

Q.72) Solution (b)

Collective Responsibility This is the bedrock principle of parliamentary government. The


ministers are collectively responsible to the Parliament in general and to the Lok Sabha in
particular (Article 75). They act as a team, and swim and sink together. The principle of
collective responsibility implies that the Lok Sabha can remove the ministry (i.e., council of
ministers headed by the prime minister) from office by passing a vote of no confidence.

Rajya Sabha cannot have a Vote of No Confidence.

www.iasbaba.com Page 60
POLITY

23. Chief Minister

24. State Council of Ministers

Q.73) Which one of the following statements is correct?

a) In India, the same person cannot be appointed as governor for two or more states at the
same time.
b) The judges of the high court of the states in India are appointed by the Governor of the
state just as the judges of the Supreme Court are appointed by the president.
c) No procedure has been laid down in the constitution of India for the selection and
appointment of the Chief Minister.
d) In the case of a union territory having a legislative setup, the chief minister is appointed
by the Lt. Governor on the basis of majority support.

Q.73) Solution (c)

In India, the same person can be appointed as governor for two or more states at the same
time.

The judges of the high court of the states in India are appointed by the President only, just as
the judges of the Supreme Court are appointed by the President.

The Constitution does not contain any specific procedure for the selection and appointment of
the Chief Minister.

Article 164 only says that the Chief Minister shall be appointed by the governor. However, this
does not imply that the governor is free to appoint any one as the Chief Minister.

 In accordance with the convections of the parliamentary system of government, the


governor has to appoint the leader of the majority party in the state legislative assembly
as the Chief Minister.
 But, when no party has a clear majority in the assembly, then the governor may exercise
his personal discretion in the selection and appointment of the Chief Minister.
 In such a situation, the governor usually appoints the leader of the largest party or
coalition in the assembly as the Chief Minister and ask him to seek a vote of confidence
in the House within a month.

www.iasbaba.com Page 61
POLITY

Chapter 25. Parliament

Q.74) Which among the following can be said to be a part of the Parliament of India?

1. President
2. Lok Sabha
3. Rajya Sabha

Select the code from below:

a) 1 only
b) 2 and 3
c) All of the above
d) 1 and 2

Q.74) Solution (c)

Direct Easy question.

Q.75) Consider the following statements in regard to representatives of UTs in the second
chamber:

1. They are indirectly elected by members of an electoral college specially constituted for
the purpose.
2. The Constitution has empowered the Parliament to prescribe the manner of choosing
the representatives of the UTs to the second chamber.

Which of the statements given above is/are correct?

a) 1 only
b) 2 only
c) Both 1 and 2
d) Neither 1 nor 2

Q.75) Solution (b)

www.iasbaba.com Page 62
POLITY

The Constitution has empowered the Parliament to prescribe the manner of choosing the
representatives of the UTs in the Lok Sabha (but not second chamber i.e., Rajya Sabha). Hence,
statement (2) is wrong.

The representatives of each union territory in the Rajya Sabha are indirectly elected by
members of an electoral college specially constituted for the purpose. This election is also held
in accordance with the system of proportional representation by means of the single
transferable vote.

Note: Out of the seven union territories, only two (Delhi and Puducherry) have representation
in Rajya Sabha. The populations of other five union territories are too small to have any
representative in the Rajya Sabha.

Q.76) Consider the following statements regarding the ‘Upper House’ of Parliament:

1. Unlike lower House, the dissolution of Upper House takes place after every 6 years.
2. According to Constitution the term of members of Rajya Sabha is fixed as 6 years.
3. The retiring members are eligible for re – election in Rajya Sabha only twice.

Which of the above statements are correct?

a) 1 and 2
b) 2 and 3
c) 3 only
d) None of the above

Q.76) Solution (d)

All the options are incorrect.

The Rajya Sabha (first constituted in 1952) is a continuing chamber, that is, it is a permanent
body and not subject to dissolution. However, one-third of its members retire every second
year. Their seats are filled up by fresh elections and presidential nominations at the beginning
of every third year.

The retiring members are eligible for re-election and renomination any number of times.

The Constitution has not fixed the term of office of members of the Rajya Sabha and left it to
the Parliament. Accordingly, the Parliament in the Representation of the People Act (1951)

www.iasbaba.com Page 63
POLITY

provided that the term of office of a member of the Rajya Sabha shall be six years. The act also
empowered the president of India to curtail the term of members chosen in the first Rajya
Sabha. In the first batch, it was decided by lottery as to who should retire. Further, the act also
authorised the President to make provisions to govern the order of retirement of the members
of the Rajya Sabha.

Q.77) Which among the given statements is/are true in regard to the Speaker of Lok Sabha?

1. He is the guardian of powers and privileges of the members, the House as a whole and
its committees.
2. He announces the government policies on the floor of the house.

Select the appropriate code

a) 1 only
b) 2 only
c) Both 1 and 2
d) Neither 1 nor 2

Q.77) Solution (a)

Speaker is the final interpreter of the provisions of (a) the Constitution of India, (b) the Rules of
Procedure and Conduct of Business of Lok Sabha, and (c) the parliamentary precedents, within
the House.

Speaker is also the guardian of powers and privileges of the members, the House as a whole
and its committees.

However, it is the Prime Minister or Chief Minister who announces the government policies on
the floor of their respective house. Hence, 2nd statements is wrong.

Q.78) Which of the following statements are correct about Pro – tem Speaker of Lok Sabha?

1. The Protem Speaker has all the powers of a Speaker.


2. He Presides over the first sitting of Newly Elected Lok Sabha.
3. He administers Oath to new members of the house.
4. He enables the house to elect a new speaker.

Select the code from below:

www.iasbaba.com Page 64
POLITY

a) 1,2 and 3
b) 2,3 and 4
c) 1,3 and 4
d) All of the above

Q.78) Solution (d)

As provided by the Constitution, the Speaker of the last Lok Sabha vacates his office
immediately before the first meeting of the newly- elected Lok Sabha. Therefore, the President
appoints a member of the Lok Sabha as the Speaker Pro Tem . Usually, the seniormost member
is selected for this. The President himself administers oath to the Speaker Pro Tem.

The Speaker Pro Tem has all the powers of the Speaker. He presides over the first sitting of the
newly-elected Lok Sabha. His main duty is to administer oath to the new members. He also
enables the House to elect the new Speaker. When the new Speaker is elected by the House,
the office of the Speaker Pro Tem ceases to exist. Hence, this office is a temporary office,
existing for a few days.

Q.79) A ‘Zero Hour’ is an Indian innovation in the field of Parliamentary procedures. Which of
the following statements are correct about the ‘Zero Hour’?

1. It is the first hour of the parliaments’ session.


2. Members of Parliament can raise questions without any prior notice.

Select the code from below:

a) 1 only
b) 2 only
c) Both 1 and 2
d) Neither 1 nor 2

Q.79) Solution (b)

The first hour of every parliamentary sitting is the question hour. During this time, the members
ask questions and the ministers usually give answers. The questions are of three kinds, namely,
starred, unstarred and short notice.

 A starred question (distinguished by an asterisk) requires an oral answer and hence


supplementary questions can follow.

www.iasbaba.com Page 65
POLITY

 An unstarred question, on the other hand, requires a written answer and hence,
supplementary questions cannot follow.

 A short notice question is one that is asked by giving a notice of less than ten days. It is
answered orally.

Unlike the question hour, the zero hour is not mentioned in the Rules of Procedure. Thus it is an
informal device available to the members of the Parliament to raise matters without any prior
notice.

The zero hour starts immediately after the question hour and lasts until the agenda for the day
(ie,regular business of the House) is taken up. In other words, the time gap between the
question hour and the agenda is known as zero hour. It is an Indian innovation in the field of
parliamentary procedures and has been in existence since 1962.

Q.80) A member of Parliament can be disqualified on the grounds of defection. On which of


the following grounds can he be disqualified?

1. If he voluntary gives up the membership of the political party on whose ticket he is


elected to the House.
2. If any independently elected member joins any political party
3. If any nominated member joins any political party after the expiry of six months.

Select the code from below:

a) 1 only
b) 2 and 3
c) 1 and 3
d) All of the above

Q.80) Solution (d)

Disqualification on Ground of Defection

The Constitution lays down that a person shall be disqualified from being a member of
Parliament if he is so disqualified on the ground of defection under the provisions of the Tenth
Schedule. A member incurs disqualification under the defection law:

www.iasbaba.com Page 66
POLITY

 if he voluntary gives up the membership of the political party on whose ticket he is


elected to the House;
 if he votes or abstains from voting in the House contrary to any direction given by his
political party;
 if any independently elected member joins any political party; and
 if any nominated member joins any political party after the expiry of six months.

The question of disqualification under the Tenth Schedule is decided by the Chairman in the
case of Rajya Sabha and Speaker in the case of Lok Sabha (and not by the president of India). In
1992, the Supreme Court ruled that the decision of the Chairman/Speaker in this regard is
subject to judicial review.

www.iasbaba.com Page 67

You might also like